Тема Формула Единства
Уравнения, системы и тождественные преобразования на ФЕ
Вспоминай формулы по каждой теме
Решай новые задачи каждый день
Вдумчиво разбирай решения
ШКОЛКОВО.
Готовиться с нами - ЛЕГКО!
Подтемы раздела формула единства
Решаем задачи

Ошибка.
Попробуйте повторить позже

Задача 1#85914

Докажите, что уравнение

5m2−-n
n2+ 3m =1

имеет бесконечно много решений в целых числах.

Источники: ФЕ-2023, 11.2 (см. www.formulo.org) | ФЕ - 2024, 11.6 (см. www.formulo.org)

Показать доказательство

Решим сначала уравнение

   2      2
5m  − n = n +3m

______________________________________________________________________________________________________________________________________________________

5m2 − 3m = n2+ n

Умножим на 4 и прибавим 1 к обеим частям, чтобы выделить полный квадрат справа:

20m2− 12m+ 1= (2n+1)2

Теперь домножим обе части на 5 и выделим полный квадрат слева:

(10m− 3)2 =5(2n+ 1)2+ 4

Сделаем замену x= 10m− 3,y = 2n +1  . У получившегося уравнения

x2− 5y2 =4

имеются решения

x= ±(F2k−1+F2k+1),y =±F2k,k≥ 0,

где Fk  — числа Фибоначчи (мы пользуемся нумерацией F0 = 0,F1 = 1,Fk+1 = Fk+ Fk−1  при всех целых k  ). На самом деле

(Fk−1+ Fk+1)2− 5F2k = 4F2k− 1+4Fk−1Fk− 4F 2k

равно     k
(−1)4  для всех k  , что легко проверить по индукции: при k= 0  это выполняется, а если  2            2      k
Fk−1+Fk−1Fk− Fk =(−1)  , то и

F2k + FkFk+1− F2k+1 =Fk2− Fk−1Fk− F2k−1 = (−1)k+1

(Можно доказать с помощью теории уравнений Пелля, что  2    2
x − 5y = 4  не имеет других решений.)

Теперь нужно найти бесконечно много x  и y  таких, для которых соответствующие     x+3
m = 10  и    y−1
n=  2  целые. Заметим, что последовательность остатков чисел Фибоначчи по модулю 10 периодична (так как пара ( Fk−1,Fk  ) может принимать конечное количество вариантов по модулю 10, а остаток следующего и предыдущего чисел Фибоначчи однозначно определяются по остаткам этой пары). Кроме того, x =F2 =1  и y =F1 +F3 = 3  подходят, они соответствуют тривиальному решению m = n= 0  . Значит, уравнение 5m2 − n =n2 +3m  имеет бесконечно много решений.

_________________________________________________________________________________________________________________________________________________________________________________

Осталось понять, что они все не могут обнулять знаменатель. Действительно, если (m,n)  — решение уравнения 5m2− n= n2+ 3m  , при котором n2+ 3m =0  , то и 5m2− n= 0  . Следовательно, 25m4 + 3m = 0  . Так как m  целое, то обязательно m = 0  (иначе ||  4||
25m  > |3m| ), а значит, и n= 0  . Остальные пары (m,n)  нам подходят.

Ошибка.
Попробуйте повторить позже

Задача 2#85909

Функции f  и g  заданы формулами

f(x)= ax +b,g(x)= bx+a,

где a  и b  — некоторые натуральные числа, причём

f(g(x))− g(f(x))= 2024

Чему могут быть равны числа a  и b  ?

Источники: ФЕ - 2024, 11.1 (см. www.formulo.org)

Показать ответ и решение

Условие равносильно выполнению равенства

a(bx+ a)+b − (b(ax+ b)+a)= 2024

 2     2
a + b− b − a =2024

(a− b)(a+ b− 1)= 2024

Поскольку 2024 =23⋅11⋅23  , и значения выражений a− b  и a +b− 1  разной чётности, второе из них положительно и больше первого, то остаётся рассмотреть только четыре варианта:

a − b  1 8 11 23
a+ b− 1  2024 253 184 88

Соответствующие пары значений ( a,b  ) таковы: (1013;1012),(131;123),(98;87),(56;33)  .

Ответ:

 (1013;1012),(131;123),(98;87),(56;33)

Ошибка.
Попробуйте повторить позже

Задача 3#68183

Решите в натуральных числах уравнение

 b        a
a + a+b =b

Источники: ФЕ-2023, 11.5 (см. www.formulo.org)

Подсказки к задаче

Подсказка 1

Давайте сначала переберем значения b. При 1 нет решений, при 2 можно найти одно решение. А дальше уже не хочется смотреть...и мы начинаем понимать тендецию, что справа будет стоять экспонента от a, а слева - многочлен...Как теперь строго описать этот феномен?

Подсказка 2

Вот давайте теперь предположим, что b хотя бы 3, а a хотя бы 2. Посмотрим на a^b + a + b. Вот пара оценок на помощь: a^b < a^b + a + b < a^b + ab < a^b +ba^(b-2). А как это еще можно оценить так, чтобы справа вышло тоже что-то в степени b?

Подсказка 3

Как (a+1/a)^b! Это проверяется с помощью бинома Ньютона. То есть, вышло что a^b < b^a < (a+1/a)^b. Те, кто рассматривал уравнение вида a^b = b^a знают, что нужно дальше делать) А кто нет, то вот что: приведите эти три функции к почти одинаковому виду, а после посмотрите на скорость роста функции.

Подсказка 4

Прологарифмируем и поделим на ab! выйдет: ln(a+1/a)/a > ln(b)/b > ln(a)/a. Рассмотрите производную функции ln(x)/x и поймете, как она себя ведет. А дальше надо подумать про самое больше число в этой цепочке неравенств...

Подсказка 5

С помощью производной ln(x)/x, можно понять, что нет решений при b ≥ a ≥ 3 и c a = 2, b ≥ 4. Теперь предлагается вот что: можно получить противоречие с тем, что самое наибольшее слагаемое в цепочке - не наибольшее. докажите, что ln(a+1/a)/a < ln(a-1)/(a-1) при a ≥ 4, также с помощью производных)

Показать ответ и решение

Если a =1  или b= 1,  то решений нет. Если b= 2,  то получим 2a = a2 +a+ 1.  При a< 5  решений нет, a= 5  подходит, а при a≥ 5  левая часть увеличивается менее чем в два раза при увеличении a  на 1.  Пусть b≥ 3.  Тогда

                              (   1)b
ba = ab+ a+ b≤ ab+ab≤ ab+ bab−2 < a+ a

Последнее неравенство следует из разложения по биному Ньютона для (   )
a +a1b.  Действительно:

(     )
 a + 1 b = ab +b⋅ab−1⋅ 1+ ...
    a               a

Значит,

(   1)b
 a +a   > ba >ab

логарифмируя и деля на ab,  получаем:

 (    )
ln-a+-1a--> lnb-> lna-
   a      b    a

Пусть f(x)= lnx.
      x  Заметим, что f(a)  убывает при a ≥3  и f(2)=f(4)  (у этой функции производная равна f′(x) = 1−lnx,
        x2  и она отрицательна при x> e.  Поэтому нет решений с a= 2,b≥4  и с b ≥a ≥3.

С другой стороны, можно проверить, что

  (   1)
ln-a+-a-< ln(a-− 1)
   a        a− 1

при a≥ 4.  Действительно, при a= 4  это

(   1)3          3  1       4
 4+ 4  = 64+ 12+ 4 + 64 < 81= 3

и производная выражения g(a)= a⋅ln (a− 1)− (a− 1)⋅ln(a+ 1a)  равна

        2                      (    )
− (a−-1)(2a-−-1)+ -a--+ln(a− 1)− ln  a+ 1  =
    a(a + 1)    a− 1                a

                                 (    )
= − a3−-a2−-a+1 +-a--+ln(a− 1)− ln  a+ 1  =
     a(a2+ 1)     a− 1                a

   (   −a2-− 2a+-1) (   --1-)    (-a2+-1)
=−  1+   a3+ a)   +  1+ a− 1  − ln a(a− 1) =

  a2+ 2a− 1   1     ( a2+ 1 )
= --a3+-a--+ a−-1 − ln a(a−-1)

Но (так как ln(1 +x)< x  при x> 0)

 (       )    (        )
ln  -a2-+1-  =ln 1+ a-+1- < -a+-1
   a(a − 1)        a2− a   a2− a

так что

 ′    ---a3−-3a---
g (a)> a(a− 1)(a2+ 1) > 0

уже при a ≥3.  Таким образом, уравнение не имеет решений при a≥ 4.

Замечание. Вместо оценки (    )
 a+ 1a b  можно использовать (a+1)b  (верную при b= 2),  тогда упрощаются вычисления, но нужно перебирать больше исключений.

Ответ:

 a =5,b= 2

Ошибка.
Попробуйте повторить позже

Задача 4#68179

Найдите сумму всех корней уравнения:

∘--2--------------  ∘--2--------------  ∘--2--------------
 2x − 2024x+ 1023131+  3x − 2025x+ 1023132+  4x − 2026x+ 1023133=

=∘x2-−-x+1-+∘2x2-− 2x+-2+ ∘3x2−-3x+-3

Источники: ФЕ-2023, 11.1 (см. www.formulo.org)

Подсказки к задаче

Подсказка 1

Хочется разбить радикалы по парам. Как связаны подкоренные выражения в одной паре?

Подсказка 2

Они отличаются на x^2-2023x+1023130. Тогда хочется написать какую-то оценку...

Подсказка 3

Если это выражение больше 0, то левая часть больше правой, если же это выражение меньше 0, то левая часть меньше (при условии существования обеих частей). Когда тогда может достигаться равенство?

Подсказка 4

Только если x^2-2023x+1023130=0. Отсюда находим x, и не забываем проверить, что выражения существуют!

Показать ответ и решение

Обозначим

      2           1 2  3
f(x)= x − x+ 1= (x −2) + 4 >0

g(x)= x2− 2023x+ 1023130=(x− 1010)(x− 1013)

Тогда уравнение имеет вид

∘ --------- ∘---------  ∘ ---------  ∘ ---- ∘----  ∘ ----
  f(x)+ g(x)+  2f(x)+g(x)+  3f(x)+ g(x)=   f(x)+  2f(x)+  3f(x)

Если какое-то значение x  является решением, то g(x)= 0,  ведь иначе левая часть больше (при g(x)> 0  ) или меньше (при g(x) <0  ) в силу монотонного возрастания функции      √-
h(t)=  t  на своей области определения.

При этом легко видеть, что все решения g(x)=0  являются и решениями исходного уравнения (будет верное тождество, при этом обе части определены в силу положительности функции f  ), то есть это не только необходимое, но и достаточное условие.

Корнями уравнения g(x)= 0  являются числа 1010  и 1013  . Их сумма равна 2023.

Ответ: 2023

Ошибка.
Попробуйте повторить позже

Задача 5#74786

Решите систему в целых числах:

{ (y2 +6)(x− 1)= y(x2+ 1)
  (2   )        ( 2  )
   x +6 (y− 1)= x y + 1

Источники: ФЕ-2022, 11.3 (см. www.formulo.org)

Подсказки к задаче

Подсказка 1

Оставлять такие скобки бессмысленно, поэтому раскроем их. Теперь у нас есть одинаковые слагаемые в обоих уравнениях, так что сразу начнем преобразовывать систему и приведем ее к удобному уравнению.

Подсказка 2

Сложим уравнения системы и начнем преобразовывать так, чтобы становилось как можно больше скобок. Совсем необязательно, чтобы все разложилось на множители.

Подсказка 3

(x-2)(x-3)+(y-2)(y-3)=0. Попробуем исследовать функцию f(x)=(t-3)(t-2) и понять, в каких случаях достигается равенство.

Показать ответ и решение

Раскроем скобки:

{ y2x− y2+ 6x− 6= yx2+y
  x2y− x2 +6y = xy2+x

Сложим эти 2 уравнения:

−x2− y2+ 6(x+y)− 12= x+ y

x2+y2− 5(x +y)+ 12= 0

(x2− 5x+ 6)+ (y2− 5y+ 6)= 0

(x− 2)(x− 3)+ (y− 2)(y− 3)= 0

Рассмотрим f(t)= (t− 2)(t− 3)  — это парабола с ветвями вверх, f(t)≥ 0 при t∈ℤ.  Тогда f(x)+f(y)≥0,  а равенство достигается только при

{
  f(x) =0
  f(y)= 0

То есть при

(  [
|||||   x = 2
|{   x = 3
|||  [
|||(   y = 2
    y = 3

Остаётся только проверить найденные решения, подставив их в изначальную систему.

Ответ:

 (2,2);(3,3);(2,3);(3,2)

Рулетка
Вы можете получить скидку в рулетке!